Which one of the following, if true, would most clearly support the argument made in passage B?

tomgbean on December 30, 2019

A vs. B

A is directly stated in the passage" First, a chef must not copy another chef's recipe . innovation exactly." This would insinuate that it is the most strongly supported....This question should have been phrased "which of the following could be strongly *inferred* from the passage?" Should we treat these types of questions as strengthen questions? Also, when the question is worded so that it directs you to find support for a given answer choice in the passage, how are we to know if it is a must be true question(as is common in RC) or an inference question? The argument made in passage B is that three social norms serve to protect the intellectual property rights of chefs in place of formal law. The question asks us to find the answer choice that best supports the argument. Actually, never mind this makes sense now. If breaking the norms can be punished, that is support that norms do in fact act as replacements for formal law. However, the fact that colleagues must credit developer of significant recipes as authors of those recipes is already stated in the passage so it does not provide support......If the question was "which one of the answer choices is most strongly supported by the passage?," then answer choice A might be the correct answer.

Reply
Create a free account to read and take part in forum discussions.

Already have an account? log in

shunhe on January 1, 2020

Hi @tomgbean,

I think you figured it out as you went along, but I'll help summarize what I think you wrote and add some of my own thoughts to give you another opinion. This is most definitely a Strengthen question. Here, the question isn't asking for something that must be true based on the passage or that is most strongly supported by the passage. Similarly, it isn't asking for something that "IS the most strongly supported." Instead, it asks for something that if true, would most clearly support the argument made in passage B. In other words, we are trying to bring in new information that will be presented in the answer choices in order to bolster arguments made in the passage. If a question is phrased "would most clearly support the argument," you can bet you're looking at a strengthen question. And this is why it's very important to read the question stem thoroughly! The difference between "what is most strongly supported" and "what, if true, would most strongly support" seems like a small one, but can completely change the interpretation of a question. "Must be true" or inference questions won't introduce new information, instead asking you to go back to the passage to dig up claims the author made.

As such, (A) does not tell us anything about the argument made in passage B. The argument in passage (B) suggests that three implicit social norms are used among chefs, and they operate similar to law-based IP systems. (A) doesn't tell us about the social norms; instead, it gives an example of something that's not a social norm, which doesn't help this argument at all. (A) could have just as easily have been, "There is no social norm preventing chefs from bringing their kids to work." And if (A) is stated directly in the passage, then it doesn't even present new information with which to work, and so CANNOT strengthen the passage because it's already contained in the passage!

(B), on the other hand, suggests that punishments exist when social norms are violated, which supports the idea that these social norms function as a law-based IP system. Hope this helps! Feel free to ask any questions if you need further clarification, though it seems like you got it.